GR9768 Q 63

Forum for the GRE subject test in mathematics.
Post Reply
mathQ
Posts: 41
Joined: Thu Mar 25, 2010 12:14 am

GR9768 Q 63

Post by mathQ » Tue Apr 06, 2010 7:50 am

Ques : At how many points in the xy-plane do the graphs of y = x^12 and y = 2^x intersect ?

A) None
B) 1
C) 2
D) 3
E) 4

I could figure out 2 points, but the ans is D.

Also any general method of solving such problems ?

Thanks.

speedychaos4
Posts: 47
Joined: Mon Mar 22, 2010 2:42 am

Re: GR9768 Q 63

Post by speedychaos4 » Tue Apr 06, 2010 8:01 am

since your ans is 2 , i guess you let x^12=2^x and use log_2 for both sides right? note that log is only defined for (0,+inf), looking back at the graph of the 2 func. you should see effortlessly that they intersect at some where when x<0.

mathQ
Posts: 41
Joined: Thu Mar 25, 2010 12:14 am

Re: GR9768 Q 63

Post by mathQ » Tue Apr 06, 2010 8:11 am

speedychaos4 wrote:since your ans is 2 , i guess you let x^12=2^x and use log_2 for both sides right? note that log is only defined for (0,+inf), looking back at the graph of the 2 func. you should see effortlessly that they intersect at some where when x<0.

No, I did not do it that way (by taking log)

I plotted the (rough) graphs.

EugeneKudashev
Posts: 27
Joined: Tue Apr 06, 2010 8:22 am

Re: GR9768 Q 63

Post by EugeneKudashev » Tue Apr 06, 2010 8:33 am

you may stop worrying right away, plotting graphs is the right thing to do. ETS just made a bloody typo, there is no third intersection point. for those who still unsure it is easy to go there > http://www.wolframalpha.com/input/?i=2^x+%3D+x^12 and see:
x ~= -0.9467803303936548
x ~= 1.063346830653734
--just what one gets with rough plotting.

speedychaos4
Posts: 47
Joined: Mon Mar 22, 2010 2:42 am

Re: GR9768 Q 63

Post by speedychaos4 » Tue Apr 06, 2010 8:49 am

Sketching roughly might be what ETS wants to mislead you into.
Since every single question in the test is very important (at least true for me), you better be 90% or 100% sure before you choose the answer.
My approach is that, by letting
x^12=2^x,
we could have
log_2(x^12)=log_2(2^x),
which yields
log_2(x)=x/12
let
g(x)=log_2(x)-x/12
then
g'(x)=1/(xln2)-1/12
note that g'(x)>0 when x is close to 0 and there is a certain point x0 from which g'(x)<0
note also g(x)<0 when x is close to 0
from g'(x) we know that for x<x0, g(x) is strictly increasing and we can easily find a point at which g(x)>0, x=2 for example, and g(x) becomes strictly decreasing when x>x0, therefore, we could know that the 2 func. intersects twice on (0,+inf)

I afraid it is too complicated, but it works for me.

EugeneKudashev
Posts: 27
Joined: Tue Apr 06, 2010 8:22 am

Re: GR9768 Q 63

Post by EugeneKudashev » Tue Apr 06, 2010 11:22 am

speedychaos4 wrote:we could know that the 2 func. intersects twice on (0,+inf)
but you see that your conclusion is wrong since the equation has only one positive root (refer to wolfram alpha)

speedychaos4
Posts: 47
Joined: Mon Mar 22, 2010 2:42 am

Re: GR9768 Q 63

Post by speedychaos4 » Tue Apr 06, 2010 11:42 am

EugeneKudashev wrote:
speedychaos4 wrote:we could know that the 2 func. intersects twice on (0,+inf)
but you see that your conclusion is wrong since the equation has only one positive root (refer to wolfram alpha)
But if you take a more numerical approach, for example, by the wolfram figure you provide, it is clear that 2^x<x^12 when x=2, then take a large no, for example, x=120, then 2^x=2^120=(2^10)^12=1024^12>120^12=x^12, therefore they must intersects somewhere on (2,120)

EugeneKudashev
Posts: 27
Joined: Tue Apr 06, 2010 8:22 am

Re: GR9768 Q 63

Post by EugeneKudashev » Tue Apr 06, 2010 12:01 pm

the latter is pretty convincing, I lay down my arms :)
now the question is, why wolfram presents only two solutions (numerically). the third root is approx 74.6, as I was able to conclude via plotting the graphs in gnuplot. thanks anyway, up to this very day I was absolutely sure that it was a typo.

enork
Posts: 33
Joined: Fri Sep 18, 2009 3:16 am

Re: GR9768 Q 63

Post by enork » Tue Apr 06, 2010 11:20 pm

Put more simply: an exponential eventually grows faster than any polynomial so 2^x will overtake x^12 at some large enough x.



Post Reply